How Do You Sketch Phase Plane Diagrams for Differential Equations?

  • Thread starter Thread starter EV33
  • Start date Start date
  • Tags Tags
    Phase Plane
Click For Summary
To sketch phase plane diagrams for differential equations, start by identifying the four asymptotic lines based on the constants C1 and C2. Each combination of positive and negative values for C1 and C2 will yield specific functions for x^(1) and x^(2). The direction of these asymptotic lines can be determined by analyzing the behavior of the solutions as t approaches infinity. To draw the various solutions, consider the initial conditions and how they influence the trajectory in the phase plane. Understanding these elements will help in accurately depicting the phase plane diagram.
EV33
Messages
192
Reaction score
0

Homework Statement



In general, how do you draw the phase plane for the given solution?

x=C1*e^{lambda1*t}[a1 a2]^{T} + C2*e^{lambda2*t}[b1 b2]^{T}

I think I know how to get the four asymptotic lines.

I am not sure how to determine the direction of my asymptotic lines
or how to draw the other solutions.

Homework Equations


C1,C2,a1,a2,b1,b2 are constants
T means transpose
t is the variable


The Attempt at a Solution



1.) So we have to draw four lines.

Let c1>0,c2=0
c1<0, c2=0
c2>0,c1=0
c2<0, c1=0

Each one of these will give you functions of x^{(1)} and x^{(2)}.
You will also be able to determine that x^{(1)} or x^{(2)} is less than/greater than 0 based off of the innequalities.

2.) I have no idea on how to determine the direction of the asymptotic lines.

3.) Also I don't know where to start my the solutions and where to end them when I draw them.



Thank you for your time.
 
Physics news on Phys.org
ok I just figured out how to get the directions of the asymptotic lines so that just leaves my last question.

How do you determine the path of the various solutions?
 
Question: A clock's minute hand has length 4 and its hour hand has length 3. What is the distance between the tips at the moment when it is increasing most rapidly?(Putnam Exam Question) Answer: Making assumption that both the hands moves at constant angular velocities, the answer is ## \sqrt{7} .## But don't you think this assumption is somewhat doubtful and wrong?

Similar threads

  • · Replies 17 ·
Replies
17
Views
3K
Replies
4
Views
2K
  • · Replies 1 ·
Replies
1
Views
2K
  • · Replies 1 ·
Replies
1
Views
2K
  • · Replies 1 ·
Replies
1
Views
2K
  • · Replies 7 ·
Replies
7
Views
2K
  • · Replies 9 ·
Replies
9
Views
2K
  • · Replies 5 ·
Replies
5
Views
2K
  • · Replies 4 ·
Replies
4
Views
2K
  • · Replies 24 ·
Replies
24
Views
3K